Tort quizzes

¡Supera tus tareas y exámenes ahora con Quizwiz!

B. The plaintiff is required to prove that the defendant should have reasonably foreseen, as a risk of her conduct, the general consequences or type of harm suffered by the plaintiff.

. In a negligence action, what is the plaintiff required to prove in order to establish proximate (or legal) causation? A. The plaintiff is required to prove that but for the defendant's breach of the standard of care, the plaintiff would not have suffered the complained of harm. B. The plaintiff is required to prove that the defendant should have reasonably foreseen, as a risk of her conduct, the general consequences or type of harm suffered by the plaintiff. C. The plaintiff is required to prove that the defendant should have reasonably foreseen, as a risk of her conduct, the general consequences of type of harm suffered by the plaintiff and the manner in which the harm came about. D. The plaintiff is required to prove that the defendant should have reasonably foreseen, as a risk of her conduct, the general consequences of type of harm suffered by the plaintiff, the extent of the harm, and the manner in which the harm came about.

B. Driver would not have hit her or would have caused her less harm if he had not been exceeding the speed limit at the time she was hit.

3. Driver is driving his car at 60 miles per hour on a stretch of highway with a 50 mile per hour speed limit when he runs into Pedestrian, seriously injuring her. In her negligence action against Driver to recover for her injuries, Pedestrian alleges that Driver was negligent in exceeding the speed limit. In order to establish factual causation (also known as causation-in-fact) as part of her negligence action, what will Pedestrian need to prove? A. Driver ran into her when he was speeding. B. Driver would not have hit her or would have caused her less harm if he had not been exceeding the speed limit at the time she was hit. C. It was foreseeable to Driver that while speeding, he would run into a person walking along the highway thereby causing physical injury. D. It was foreseeable to Driver that while speeding he would run into Pedestrian, causing her injuries.

D. If Lincoln can prove both that Dow Aircraft providing the omitted instructions and a mechanic properly consulting the service manual would have prevented the damage to Lincoln's plane, Dow Aircraft's multiple acts of negligence are a factual cause of the plane's damage.

4. Lincoln's airplane was seriously damaged when he was forced to land it without its retractable gear in the down position. Lincoln sues Dow Aircraft, the manufacturer of the plane. Lincoln claims that Dow Aircraft neglected to include instructions in its service manual of the need to be sure of a minimum clearance between two parts in the landing gear assembly when reassembling the gear after routine servicing. Lincoln's plane was serviced at Dow because Dow needed to complete unrelated warranty work to the aircraft. The Dow mechanic who worked on Lincoln's plane was fired shortly after the work was completed for repeatedly failing to consult service manuals. In a cause of action against Dow Aircraft and the mechanic, Lincoln includes a claim based on the mechanic's negligence. Which of the following statements regarding factual cause of the harm to Lincoln's plane is correct? A. The omitted instruction by itself was a factual cause of the harm to Lincoln's plane. B. The mechanic's negligence in failing to consult the manual was by itself a factual cause of the harm to Lincoln's plane. C. Because neither the missing instructions nor the mechanic's failure to consult the manual were a factual cause of Lincoln's harm, Lincoln will not be able to establish that Dow Aircraft was a factual cause of the damage to his plane. D. If Lincoln can prove both that Dow Aircraft providing the omitted instructions and a mechanic properly consulting the service manual would have prevented the damage to Lincoln's plane, Dow Aircraft's multiple acts of negligence are a factual cause of the plane's damage.

B. Pool Co. cannot be held liable to Mildred for her injuries because Mildred has failed to establish a defect necessary to impose liability on Pool Co. caused by its defective product.

8 Pool Co. manufactures aboveground swimming pools for residential use. Because the pools are only four feet deep, Pool Co. has posted warnings in large letters on the outside of their pools that warn "DO NOT DIVE INTO POOL. SEVERE HEAD INJURY RISK." While in a friend's backyard party, Mildred, age 25, reads the warning and chooses to dive head first into a Pool Co. pool. In doing so, she strikes her head on the bottom of the pool, suffering a severe head and spine injury. At trial in the tort action brought by Mildred against Pool Co. to recover for her injuries, experts testify that when Mildred's hands hit the pool's slippery vinyl bottom, her hands slid apart causing her head to strike the bottom of the pool very hard. Experts testify that the pool was designed to do so in order that people who may dive in would be guided away from the pool's bottom. The testimony is that this is the safest way to make an aboveground swimming pool. Experts also testify that the pool in which she was injured did not deviate from its intended design. Expert witnesses for both sides also agree that the vinyl pool liner used in the pool in which Mildred was injured was the best and safest liner available at the time and that no alternative, less slippery liner was feasible. At trial, the factfinder determines that the warnings were adequate. Should Pool Co. be held liable to Mildred? A. Pool Co. may be held liable to Mildred for her injuries because Mildred has established that its pool had a manufacturing defect that caused her injury. B. Pool Co. cannot be held liable to Mildred for her injuries because Mildred has failed to establish a defect necessary to impose liability on Pool Co. caused by its defective product. C. Pool Co. may be held liable to Mildred for her injuries because she was injured in one of its pools. D. Pool Co. may be held liable to Mildred because she was not a foreseeable user of the pool.

D. If the jury determines that Scientist's actions of distracting the gorilla amounted to assumption of the risk, such assumption of the risk will reduce Scientist's recovery by the amount of fault attributed to Scientist.

A 328-pound gorilla escapes from its enclosure in City Zoo and manages to escape from zoo premises. A local animal scientist, Scientist, well familiar with the dangers presented by an adult gorilla, sees the gorilla wandering through the scientist's neighborhood, and she tries to distract the beast so that a group of small children playing nearby will not be injured. In doing so, Scientist is seriously injured. If Scientist sues City zoo in strict liability in a jurisdiction that has adopted a pure comparative fault scheme, which of the following statements regarding the strict liability of City Zoo is correct? A. If the jury determines that Scientist's actions of distracting the gorilla amounted to assumption of the risk, such assumption of the risk will completely bar Scientist from recovering. B. City Zoo may not be subject to strict liability for injuries from its animals because of the benefits that a zoo provides to the public. C. City Zoo may not be subject to strict liability for Scientist's injuries because they occurred away from the zoo's premises. D. If the jury determines that Scientist's actions of distracting the gorilla amounted to assumption of the risk, such assumption of the risk will reduce Scientist's recovery by the amount of fault attributed to Scientist.

D. Apartment Owner is subject to vicarious liability for Tenant's injuries because Apartment Owner hired Ivan for an activity for which a city ordinance imposed an obligation on Apartment Owner and Ivan's failure to comply with the city ordinance was a factual cause of Tenant's injury.

A city ordinance requires that all apartments be equipped with fire sprinklers in each sleeping area and in each stairwell leading to an occupied area. Apartment Owner, who owns an apartment building and leases apartments, hires Ivan, an independent electrical and plumbing contractor, to install fire sprinklers throughout the building. Ivan does not install a fire sprinkler in every stairwell as specified by the ordinance. Tenant, a tenant in Apartment Owner's building, is injured in a fire that started in a stairwell without sprinklers. Tenant would have escaped the fire without injury if a fire sprinkler had been in place. In a lawsuit brought by Tenant against Apartment Owner for injuries sustained in the fire, may Apartment Owner be subject to vicarious liability? A. Apartment Owner may not be subject to vicarious liability for Tenant's injuries because Ivan was an independent contractor. B. Apartment Owner may not be subject to vicarious liability to Tenant for Ivan's conduct unless Apartment Owner was negligent in hiring Ivan. C. Apartment Owner may be subject to vicarious liability because he hired Ivan. D. Apartment Owner is subject to vicarious liability for Tenant's injuries because Apartment Owner hired Ivan for an activity for which a city ordinance imposed an obligation on Apartment Owner and Ivan's failure to comply with the city ordinance was a factual cause of Tenant's injury.

false

A key goal of tort law is to deter all civil harms. True False

D. B, C, D and E may all maintain actions against A

A pointed a gun at B scaring B and without authority or justification, A ordered B into a room and locked the door for 2 hours. B was awake the whole time and worried about what would happen until A released him unharmed. When A initially pointed gun at B, B was standing next to C and C dove for cover thinking the gun was pointed at her. In diving for cover C collided violently with D causing D to spill coffee on E's computer. E subsequently had to pay to have the computer repaired. Who may maintain an action against A? A. Only B but not C, D and E may maintain an action against A. B. Only B and C but not D and E may maintain an action against A C. Only B, C and D but not E may maintain an action against A D. B, C, D and E may all maintain actions against A

B. Driver should not be permitted to use the regulation's violation to establish Railway's negligence because the prevention of personal injuries is not part of the regulation's purpose.

A state administrative regulation prohibits railroad trains from blocking highway crossings for more than 10 minutes. Railway allows one of its trains to remain in a highway crossing for 30 minutes. Fifteen minutes into this 30-minute period, Driver, who is driving his car on the highway, fails to notice the train until it is too late and collides with it, suffering an injury. Driver sues Railway in negligence. At trial, Driver concedes that his negligence should reduce his recovery under comparative negligence principles, but claims that Railway is negligent per se for violating the regulation. The evidence at trial reveals that the history of the regulation and of the agency's findings accompany the regulation shows that the only purpose of the regulation is to encourage the free flow of traffic and prevent traffic delays. What effect, if any, does Railroad's violation of the regulation have on Railroad's liability for negligence? A. Driver should be able to use evidence of the regulation's violation to establish that Railway's negligence because it is a state administrative regulation applicable to railroad trains B. Driver should not be permitted to use the regulation's violation to establish Railway's negligence because the prevention of personal injuries is not part of the regulation's purpose. C. Driver may not use the regulation's violation as proof of Railway's negligence per se because Driver has conceded his own negligence. D. Driver should not be permitted to use evidence of the regulation's violation to establish that Railway was negligent because administrative regulations may not be used to establish the standard of care in a negligence action.

C. Passenger will likely not be able to use the statute to prove Driver's negligence because the statute was designed to prevent accidents between cars moving in the same direction, not to prevent accidents between cars moving in opposite directions.

A state statute requires that all slow-moving vehicles drive as far to the right on the highway as possible. The statute's primary purpose is to minimize the safety hazards posed by vehicles whose slow speed can interfere with the traffic flow. Driver is driving slowly on a highway heading north and is violating the statute by driving in the fast lane on the left. Passenger is riding along in her car. Unexpectedly, a car heading south on the highway crosses the median line striking Driver's car and injuring Passenger. Had Driver been in the lane on the far right, her car would have avoided contact with the other car. Passenger sues Driver, seeking to establish her negligence by proving her violation of the statute. What effect, if any, may proof of Driver's violation of the statute have in a negligence action brought by Passenger? A. Driver's violation of the statute is negligence per se because the purpose of the statute is to protect highway safety. B. Driver's violation of the statute may be used to establish the standard of care because any statute may be used to establish the standard of care in a negligence action. C. Passenger will likely not be able to use the statute to prove Driver's negligence because the statute was designed to prevent accidents between cars moving in the same direction, not to prevent accidents between cars moving in opposite directions. D. Passenger will likely be able to use the statute to establish the standard of care in negligence because if she had not been violating the statute, Passenger would not have been injured.

A. Chuck's violation of the statute may be used to establish his negligence, because the basic purpose of the statute is to prevent accidents of this type and Alice is in the group of persons the statute is designed to protect.

A state statute requires that the operator of a truck that becomes disabled on a highway promptly put out a warning sign at least 100 feet behind the truck. When a deflated tire disables Chuck's truck, he promptly places out a warning sign right next to the truck rather than at the 100-foot distance. Alice, approaching Chuck's truck from behind, does not see Chuck's warning sign until it is too late for her to stop. Her car strikes the rear of Chuck's truck, and she is injured in the collision. Alice sues Chuck in negligence, and the evidence at trial shows that Alice would have been able to stop in time had the warning sign been set at the 100-foot distance as required by statute. What effect, if any, does Chuck's violation of the statute have on his liability for negligence? A. Chuck's violation of the statute may be used to establish his negligence, because the basic purpose of the statute is to prevent accidents of this type and Alice is in the group of persons the statute is designed to protect. B. Chuck's violation of the statute is evidence of res ipsa loquitur because the basic purpose of the statute is to prevent accidents of this type and Alice is in the group of persons the statute is designed to protect. C. Chuck's violation of the statute has no effect on his liability for negligence because the statute cannot be used to establish the standard of care in Alice's negligence action against Chuck because the statute was not designed expressly to provide a private right of action in negligence D. Chuck's violation of the statute has no effect on his liability for negligence because the standard of care in negligence must be established by proving what the reasonable prudent person in the same or similar circumstances would have done.

A. Yes. That's creepy, and she would've been offended if she knew.

A woman falls asleep on a train from Atlanta to Washington DC. A stranger walks over and kisses her, but she does not wake up. She doesn't find out about the kiss until later. Is this a battery? A. Yes. That's creepy, and she would've been offended if she knew. B. No. She had no idea it happened, so there was no way she could've been offended.

C. Abe's own negligence of being intoxicated does not affect Abe's recovery because Abe's intoxication was not a factual cause of his own injuries.

Abe, who is negligently driving while intoxicated, is stopped at a red light. Betty negligently fails to stop and hits Abe's car in a rear-end collision. Abe sues Betty in negligence for personal injuries he suffered in the collision. In this comparative negligence jurisdiction, what effect will Abe's negligence have on his recovery? A. Abe cannot recover in negligence from Betty because he was intoxicated at the time. B. Abe's own negligence of being intoxicated will affect his recovery because his intoxication was a factual cause of his own injuries. C. Abe's own negligence of being intoxicated does not affect Abe's recovery because Abe's intoxication was not a factual cause of his own injuries. D. Abe's own negligence of being intoxicated will affect his recovery because his intoxication was a proximate cause of his own injuries.

A. Client will have to establish, by introducing expert testimony, that Attorney Matthew failed to perform as the reasonable prudent attorney in handling Client's medical malpractice action against Ophthalmologist.

About two years ago, Client was a patient of Ophthalmologist. Client alleges that during the time that he was Ophthalmologist's patient, Ophthalmologist improperly performed LASIK surgery on Client's eyes. The LASIK surgery was supposed to correct Client's poor vision. Instead, the surgery was performed skillfully, but resulted in blindness in both eyes, a risk about which Client was never informed and a reasonable person would not have known. If Client had known of the risk of blindness, he would not have opted for the surgery. Client hires Attorney Matthew to sue Ophthalmologist for medical malpractice. Attorney Matthew fails to file the complaint to initiate the cause of action against Ophthalmologist within the applicable statute of limitations. As a result, the court dismisses Client's case against Ophthalmologist. Client then hires Attorney Becky to represent him in a cause of action against Attorney Matthew for legal malpractice. In the legal malpractice action brought by Client against Attorney Matthew, how will the professional standard of care owed by Attorney Matthew need to be established? A. Client will have to establish, by introducing expert testimony, that Attorney Matthew failed to perform as the reasonable prudent attorney in handling Client's medical malpractice action against Ophthalmologist. B. Client will be able to establish that Attorney Matthew failed to perform as the reasonable prudent attorney in handling Client's medical malpractice action without expert testimony regarding the standard of care. C. Client will be able to establish the standard of care in the legal malpractice action against Attorney Matthew by applying a rule of law. D. The factfinder will determine the standard of care in this legal malpractice action without evidentiary guidance.

C. To prove factual causation in the legal malpractice action, Client will need to prove that he more likely than not would have prevailed in the underlying medical malpractice action.

About two years ago, Client was a patient of Ophthalmologist. Client alleges that during the time that he was Ophthalmologist's patient, Ophthalmologist improperly performed LASIK surgery on Client's eyes. The LASIK surgery was supposed to correct Client's poor vision. Instead, the surgery was performed skillfully, but resulted in blindness in both eyes, a risk about which Client was never informed and a reasonable person would not have known. If Client had known of the risk of blindness, he would not have opted for the surgery. Client hires Attorney Matthew to sue Ophthalmologist for medical malpractice. Attorney Matthew fails to file the complaint to initiate the cause of action against Ophthalmologist within the applicable statute of limitations. As a result, the court dismisses Client's case against Ophthalmologist. Client then hires Attorney Becky to represent him in a cause of action against Attorney Matthew for legal malpractice. In the legal malpractice action brought by Client against Attorney Matthew, how, if at all, must Client prove factual causation? A. Client will not need to prove factual causation in this legal malpractice action. B. To prove factual causation in the legal malpractice action, Client will need to prove that he possibly would have prevailed in the underlying medical malpractice action. C. To prove factual causation in the legal malpractice action, Client will need to prove that he more likely than not would have prevailed in the underlying medical malpractice action. D. To prove factual causation in the legal malpractice action, Client will need to prove that Attorney Matthew breached the standard of care in not filing the complaint before the statute of limitations ran.

B. Attorney Matthew had a duty to act as the reasonable prudent lawyer under the same or similar circumstances.

About two years ago, Client was a patient of Ophthalmologist. Client alleges that during the time that he was Ophthalmologist's patient, Ophthalmologist improperly performed LASIK surgery on Client's eyes. The LASIK surgery was supposed to correct Client's poor vision. Instead, the surgery was performed skillfully, but resulted in blindness in both eyes, a risk about which Client was never informed and a reasonable person would not have known. If Client had known of the risk of blindness, he would not have opted for the surgery. Client hires Attorney Matthew to sue Ophthalmologist for medical malpractice. Attorney Matthew fails to file the complaint to initiate the cause of action against Ophthalmologist within the applicable statute of limitations. As a result, the court dismisses Client's case against Ophthalmologist. Client then hires Attorney Becky to represent him in a cause of action against Attorney Matthew for legal malpractice. In the legal malpractice action brought by Client against Attorney Matthew, what duty applies to Attorney Matthew? A. Attorney Matthew had a duty to act as the average lawyer under the same or similar circumstances. B. Attorney Matthew had a duty to act as the reasonable prudent lawyer under the same or similar circumstances. C. Attorney Matthew had a duty to act as the reasonable prudent lawyer in the same or similar locality. D. Attorney Matthew had a duty to act as the reasonable lawyer with Attorney Matthew's same training and experience.

D. Ophthalmologist's failure to inform Client of the material risks and available alternatives to LASIK treatment is a breach of the standard of care in negligence.

About two years ago, Client was a patient of Ophthalmologist. Client alleges that during the time that he was Ophthalmologist's patient, Ophthalmologist improperly performed LASIK surgery on Client's eyes. The LASIK surgery was supposed to correct Client's poor vision. Instead, the surgery was performed skillfully, but resulted in blindness in both eyes, a risk about which Client was never informed and a reasonable person would not have known. If Client had known of the risk of blindness, he would not have opted for the surgery. Client hires Attorney Matthew to sue Ophthalmologist for medical malpractice. Regarding the underlying medical malpractice action, which of the following statements concerning the Ophthalmologist's liability is correct? A. Ophthalmologist's failure to inform Client of the material risks and available alternatives to LASIK treatment does not constitute a breach of the standard of care in negligence because every patient should know that any eye surgery could lead to blindness. B. Ophthalmologist's failure to inform Client of the material risks and available alternatives to LASIK treatment does not constitute a breach of the standard of care in negligence because the reasonable patient should know that any eye surgery could result in blindness. C. Ophthalmologist cannot be subject to liability in negligence because he performed the surgery skillfully. D. Ophthalmologist's failure to inform Client of the material risks and available alternatives to LASIK treatment is a breach of the standard of care in negligence.

(b) II and III, only.

After a long night of hard drinking, Debra mistakenly parked her car in the driveway of her neighbor, Paul. He found, the following morning, that he could not pull his car out of his garage and into the street. It took him about an hour to rouse Debra into removing her car. Consider the following torts: I. False imprisonment II. Trespass to land III. Trespass to chattels Which did Debra commit against Paul? (a) I and II, only. (b) II and III, only. (c) I, II, and III. (d) None, because she did not intend to park in the wrong place.

A. Alice is entitled to recover $60,000.

Alice sues Bill alleging negligence. Bill defends, claiming that Alice's negligence also contributed to her injuries. At trial, the factfinder assigns 40% responsibility to Alice and 60% responsibility to Bill. The factfinder also finds that Alice's damages are $100,000. In a pure comparative fault jurisdiction, how much is Alice entitled to recover? A. Alice is entitled to recover $60,000. B. Alice is entitled to recover $100,000. C. Alice is not entitled to recover anything because she was partially responsible. D. Alice is entitled to recover $40,000.

B. Alice is entitled to recover $50,000 from Bill because her recoverable damages are her damages reduced by the amount of fault attributable to her.

Alice sues Bill alleging negligence. Bill defends, claiming that Alice's negligence also contributed to her injuries. At trial, the factfinder assigns 50% responsibility to Alice and 50% responsibility to Bill. The factfinder also finds that Alice's damages are $100,000. In a modified comparative fault (51% or "not greater than") jurisdiction, what amount, if any, is Alice entitled to recover? A. Alice is entitled to recover $50,000 from Bill because Bill only has to pay for his percentage of the judgment attributable to his fault. B. Alice is entitled to recover $50,000 from Bill because her recoverable damages are her damages reduced by the amount of fault attributable to her. C. Alice is entitled to recover $100,000 from Bill. D. Alice is barred from recovering from Bill because she was partially at fault.

D. Alice is completely barred from recovering from Bill because she was 50% at fault.

Alice sues Bill alleging negligence. Bill defends, claiming that Alice's negligence also contributed to her injuries. At trial, the factfinder assigns 50% responsibility to both Alice and Bill. The factfinder also finds that Alice's damages are $100,000. In a modified comparative fault (50% or "not as great as") jurisdiction, what amount, if any, is Alice entitle to recover from Bill? A. Alice is entitled to recover $50,000 from Bill because Bill only has to pay for his percentage of her damages attributable to his fault. B. Alice is entitled to recover $50,000 from Bill because her recoverable damages are the judgment reduced by the amount of fault attributable to her. C. Alice is entitled to recover $100,000 from Bill. D. Alice is completely barred from recovering from Bill because she was 50% at fault.

D. Alice is completely barred from recovering from Bill because she was partially at fault.

Alice sues Bill alleging negligence. Bill defends, claiming that Alice's negligence also contributed to her injuries. At trial, the factfinder assigns 50% responsibility to both Alice and Bill. The factfinder also finds that Alice's damages are $100,000. In a traditional contributory negligence jurisdiction, what amount, if any, is Alice entitled to recover? A. Alice is entitled to recover $50,000 from Bill because Bill only has to pay for his percentage of the judgment attributable to his fault. B. Alice is entitled to recover $50,000 from Bill because her recoverable damages are her damages reduced by the amount of fault attributable to her. C. Alice is entitled to recover $100,000 from Bill. D. Alice is completely barred from recovering from Bill because she was partially at fault.

B. Alice is entitled to recover $50,000 from Bill because her recoverable damages are her damages reduced by the amount of fault attributable to her.

Alice sues Bill alleging negligence. Bill defends, claiming that Alice's negligence also contributed to her injuries. At trial, the factfinder assigns 50% responsibility to both Alice and Bill. The factfinder also finds that Alice's damages are $100,000. In this pure comparative fault jurisdiction, what amount, if any, is Alice entitled to recover? A. Alice is entitled to recover $50,000 from Bill because Bill only has to pay for his percentage of the judgment attributable to his fault. B. Alice is entitled to recover $50,000 from Bill because her recoverable damages are her damages reduced by the amount of fault attributable to her. C. Alice is entitled to recover $100,000 from Bill. D. Alice is completely barred from recovering from Bill because she was partially at fault.

D. Alice, Bill, and Charles are each a factual cause of the destruction of Polly's car because their combined force resulted in the destruction Polly's car.

Alice, Bill, and Charles, each acting independently but simultaneously, negligently lean on Polly's car, which is parked at an overlook at the edge of a mountain. Their combined force results in Polly's car rolling over the edge of the cliff and plummeting down the mountain to its destruction. The force exerted by either Alice, Bill, or Charles would have been insufficient alone to propel Polly's car over the edge, but the combined force of any two of them would have been sufficient. In a lawsuit brought by Polly against Alice, Bill, and Charles based on negligence, who is the factual cause of the destruction of Polly's car? A.Neither Alice, Bill, nor Charles is a factual cause of the destruction of Polly's car because the force exerted by each of them alone would have been insufficient to move Polly's car down the mountain. B. Neither Alice, Bill, nor Charles is a factual cause of the destruction of Polly's car because the combined force of any two of them would have been sufficient. C. Two and only two of Alice, Bill, and Charles are a factual cause of the destruction of Polly's car because the combined force of any two of them would have been sufficient. D. Alice, Bill, and Charles are each a factual cause of the destruction of Polly's car because their combined force resulted in the destruction Polly's car.

C. The factfinder is likely to determine that Art is a proximate cause of Sarah's harm as well because the risk of someone falling in the parking lot due to darkness is among the risks that made Art negligent for failing to provide lighting.

Art owns a convenience store located within an exclusive, gated community. Sarah, who lives in the community, tripped on a curb in the store's parking lot one night after emerging from the store, suffering a skull fracture in the fall. Sarah sues Art for negligence in failing to have lights illuminating the parking lot, which without the lights is pitch dark. If the factfinder determines that Arthur's failure to light the parking lot was negligent and a factual cause of Sarah's harm, which of the following statements about causation is correct? A. Sarah has established the causation element of the negligence action against Arthur. B. The factfinder is likely to determine that Art's negligence is not a proximate cause of Sarah's harm because the risk of someone falling in the parking lot due to darkness is not among the risks that made Art negligent for failing to provide lighting. C. The factfinder is likely to determine that Art is a proximate cause of Sarah's harm as well because the risk of someone falling in the parking lot due to darkness is among the risks that made Art negligent for failing to provide lighting. D. The court must grant judgment in favor of Sarah.

D. Joiner can sue for assault and will be successful.

As a joke, Annette removes the bullets from her father's revolver, takes the gun outside, and points it at the head of her neighbor, Mrs. Joiner, who is just leaving her house. Mrs. Joiner, who unknown to Annette suffers from serious heart disease immediately has a stroke resulting in significant permanent damage. Which of the following best describes the most likely outcome? A. It was a joke that resulted in a tragedy, but tragically there isn't a viable intentional torts lawsuit for Joiner to pursue. B. Joiner can sue for assault, but it won't be successful because it was a joke and so it's a conditional threat. C. Joiner can sue for intentional infliction of emotional distress and will be successful as this is clearly an extreme and outrageous act. D. Joiner can sue for assault and will be successful.

C. II and IV only

Assault allows someone to recover damages, even when they have not suffered any physical harm. Which of the following statements is most accurate? i. It's an absurd rule. Why should people get money when they haven't suffered any injury? ii. It helps deter behavior that society has deemed undesirable. iii. It helps reduce lawsuits. iv. It helps provide compensation to individuals who may have suffered emotional trauma. A. I only. B. II and III only. C. II and IV only D. II, III, and IV.

A. Proximate causation determinations in negligence actions establish whether to shield a defendant from liability even though the defendant's breach of the standard of care was a factual cause of the plaintiff's harm

Which of the following statements about factual and proximate causation is correct? A. Proximate causation determinations in negligence actions establish whether to shield a defendant from liability even though the defendant's breach of the standard of care was a factual cause of the plaintiff's harm B. Proximate causation determinations are unique to negligence actions and are not made in intentional torts or strict liability actions. C. Legal causation and factual causation are synonymous terms. D. Factual causation and proximate causation are synonymous terms.

C. The differences in advantages and disadvantages of the wrap-around vest and the front-and-back only vest are sufficiently understood by consumers does not render the front-and-back design in a condition unreasonably safe.

BPV Co. manufactures bulletproof vests for use by police and security personnel. BPV Co. offers several different models, some providing front and back protection only, and others providing wrap-around protection. State highway patrol officials could and did choose to purchase the model that provides front and back protection only. They did so because that model is less expensive, allows greater flexibility of movement, and is more comfortable. State Trooper was shot and killed while making a routine traffic stop. The bullet entered the side of his body, where the vest did not provide protection. In a lawsuit brought on behalf of State Trooper's estate against BPV for injuries caused by an allegedly defective product, the estate's lawyer argues that the design of the vest is defective because it does not provide wrap around coverage. In the products liability lawsuit against BPV for State Trooper's death, what is the legal relevance of the alternative of the wrap around vest design? A. Without regard to the wrap around vest design, estate cannot recover from BPV because State Trooper willingly wore the vest with the front and back design. B. The availability of the wrap-around vest renders the vest that State Trooper was wearing, providing only front and back protection, unreasonably safe and therefore defective in design. C. The differences in advantages and disadvantages of the wrap-around vest and the front-and-back only vest are sufficiently understood by consumers does not render the front-and-back design in a condition unreasonably safe. D. In determining whether the front and back vest is defectively designed, it is legally irrelevant that the wrap-around vest is somewhat safer, is more costly to buy, and more costly to use.

B. It doesn't change the result.

Back to Garratt v. Dailey, what if Brian admits that he intended to pull a prank but said "I didn't think she'd get hurt. I never in my wildest dreams thought she'd fracture her hip. I'm only 5 years old, I don't even know what a hip fracture is. I never wanted to hurt her." A. Gamechanger. He didn't want to hurt her, so he didn't have the requisite intent. B. It doesn't change the result.

D. Billy may not be subject to strict liability for Joan's injuries because the harm that Joan suffered was not part of the risk underlying the rule for strict liability for non-domesticated animals.

Billy has a pet goat, which he keeps on his property in a carefully constructed enclosure. Billy uses all due care to make sure that his goat does not escape, but one evening, Billy's goat does get loose and wanders onto Joan's property. It is dark as the goat walks normally around Joan's property. Joan is also out taking a stroll. She does not see the goat, stumbles over it, and falls, suffering an injury. In a lawsuit brought by Joan against Billy for her injuries when she tripped over Billy's goat, may Billy be subject to liability? A. Billy may be subject to strict liability for the injuries caused by his goat because his goat caused physical harm to Joan. B. Billy may be subject to liability in negligence for failing to take reasonable precautions to make sure his goat did not escape. C. Even if Billy did not use due care in keeping his goat confined, Billy may not be subject to liability in negligence for Joan's injuries because his goat is a non-domesticated animal and therefore the only appropriate recovery is in strict liability. D. Billy may not be subject to strict liability for Joan's injuries because the harm that Joan suffered was not part of the risk underlying the rule for strict liability for non-domesticated animals.

C. Blasting Co. should not subject to strict liability for the injuries to Kylie's mink farm because the destruction of mink kittens by their mothers is not the type of damage that makes blasting an abnormally dangerous activity.

Blasting Co. is in the business of razing buildings for districts that are building up their communities. Blasting Co. is aware that there are dangers inherent in using dynamite to raze buildings, and although it adopts all reasonable precautions, it is likely that there will be some unintended damages caused by way of flying debris or the concussion of the blast. Blasting Co. has been employed by City to raze several buildings near downtown. Several miles away, Kylie has been farming minks for their fur. Unfortunately, mink are very sensitive and have very nervous dispositions, and after feeling the vibration of the blasting from many miles away (which vibrations were not detectable to humans), several of Kylie's female minks that had just given birth ate their offspring. Kylie sues Blasting Co. in strict liability for the loss of the baby minks. Should Blasting Co. be subject to strict liability? A. Blasting Co. should be subject to strict liability for the injuries to Kylie's mink farm because Blasting Co. was engaged in an abnormally dangerous activity. B. Blasting Co. should be liable in negligence to Kylie because its blasting of dynamite was a factual cause of the injuries to Kylie's mink farm. C. Blasting Co. should not subject to strict liability for the injuries to Kylie's mink farm because the destruction of mink kittens by their mothers is not the type of damage that makes blasting an abnormally dangerous activity. D. Blasting Co. should not be subject to strict liability because the blasting was not a factual cause of the destruction of the mink kittens.

D. Blasting Co. may be held strictly liable for the injuries to Driver's car because Blasting Co. was engaged in an abnormally dangerous activity.

Blasting Co. is in the business of razing buildings for districts that are building up their communities. Blasting Co. is aware that there are dangers inherent in using dynamite to raze buildings, and although it adopts all reasonable precautions, it is likely that there will be some unintended damages caused by way of flying debris or the concussion of the blast. Blasting Co. has been employed by City to raze several buildings near downtown. Unfortunately, during this particular blast, even though Blasting Co. used all of its routine precautions, Driver's car was damaged by flying debris. Driver sues Blasting Co., alleging that it should be strictly liable for the damage to Driver's car. Should Blasting Co. be held liable? A. Blasting Co. should not be held strictly liable for injuries to Driver's car because it used all due care to prevent any damage from flying debris. B. Blasting Co. should be subject to liability in negligence because it used all due care and Driver's car was still damaged. C. Blasting Co. should not subject to strict liability because Driver suffered property damage through no fault of its own. D. Blasting Co. may be held strictly liable for the injuries to Driver's car because Blasting Co. was engaged in an abnormally dangerous activity.

D. Carl may recover for Doctor's negligence for the part of Carl's injury attributable to Doctor's negligence notwithstanding the fact that Carl's negligence produced the very condition Doctor undertook to treat.

Carl negligently injures himself in an automobile accident. Carl seeks medical treatment from Doctor, who negligently aggravates Carl's injury. In a suit in which Carl seeks to recover from Doctor for the part of Carl's injuries caused by Doctor's medical malpractice, will Carl be able to recover from Doctor? A. Carl cannot recover from Doctor because Carl's negligence caused the automobile accident in which he was initially injured. B. Carl may hold Doctor liable for all of his injuries, including the injuries he sustained in the accident Carl caused. C. Carl may hold Doctor liable only if Doctor intended to cause Carl harm in providing him with medical care. D. Carl may recover for Doctor's negligence for the part of Carl's injury attributable to Doctor's negligence notwithstanding the fact that Carl's negligence produced the very condition Doctor undertook to treat.

A. Chair Palace will likely not be held liable based upon a defective design of the chair because College Student's misuse of the product is so unreasonable that the risks it entails need not be designed against.

Chair Palace manufactures various types of chairs, including an oak chair having five horizontal wooden bars that ergonomically cradle the human back. College Student has one of these oak chairs in his dorm room. Needing a book on a shelf out of his reach, College Student stands on the top bar of the chair in order to reach the book. The chair tips, and College Student falls, suffering serious harm. In a products liability lawsuit brought by College Student against Chair Palace, College Student alleges that the chair is defectively designed because it should have had the stability to support him while standing on its top bar or it should have had a differently designed back so that he could not have stood on the bar. Is Chair Place likely to be held liable? A. Chair Palace will likely not be held liable based upon a defective design of the chair because College Student's misuse of the product is so unreasonable that the risks it entails need not be designed against. B. Chair Palace will likely be subject to liability based on the chair's defective design because College Student was injured by the chair. C. Chair Palace will likely not be liable in tort because College Student was not a foreseeable user of its product. D. Chair Palace will likely be subject to liability based on a defective design because it is foreseeable that a college student will misuse a chair in this fashion.

A. Heights Hotel's departure from this custom is evidence of its negligence.

Charles is a guest in Heights Hotel. The bathroom in the hotel includes a shower, which is protected by a sliding door made of ordinary glass. While taking a shower, Charles trips and falls on the glass door, causing it to shatter. The shards of glass cut Charles causing serious injuries. It is a standard practice among hotels to use shatterproof, safety glass rather than ordinary glass at shower enclosures. In a negligence action brought by Charles against Heights Hotel, what effect, if any, will Hotel's departure from the industry custom have on Hotel's negligence? A. Heights Hotel's departure from this custom is evidence of its negligence. B. Heights Hotel's departure from this custom is conclusive proof of its negligence. C. Evidence of Heights Hotel's departure from this industry custom is legally irrelevant to the issue of its negligence. D. Despite evidence of that Heights Hotel disregarded custom, Heights Hotel may not be subject to liability in negligence because Charles tripped and fell, causing his injuries.

E. Christopher might be held liable if it turns out that he knew of the danger and could have taken reasonable precautions to avoid the harm.

Christopher owns a private home surrounded by a 10-foot fence. One night, Shawn jumps the fence and is looking around Christopher's property, when he trips over garden shears Christopher forgot to put away. Shawn breaks his ankle. So he sues Christopher. Under the traditional common law rule for landowners, what's the most likely result: A. Christopher breached his duty to keep his property clear of sharp, dangerous objects, so he will be held liable. B. Christopher will not be liable because there's no proximate cause. It's foreseeable that someone could be cut from shears, but not trip over them. C. Christopher will be held liable because a reasonable person would not leave sharp objects lying around their property in the dark. D. Christopher will not be held liable because there is no evidence of willful, wanton, or intentional conduct aimed at harming Shawn. E. Christopher might be held liable if it turns out that he knew of the danger and could have taken reasonable precautions to avoid the harm.

D. Company should be held liable in negligence because one of the risks that made Company negligent was that Harry would drive poorly and cause an accident.

Company provides a company-owned car to Harry for personal and business use. While Harry is on vacation, Harry is speeding while driving the car. He loses control, destroying a roadside billboard owned by Announcements, Inc. Announcements sues Company, claiming that Company negligently entrusted the car to Harry. Announcements claims that despite knowing of Harry's history of speeding, other moving violations, and a number of accidents, Company provided Harry with the car. Should Company be held liable in negligence for the destruction of Announcement's billboard? A. Company should not be held liable to Announcements for Harry's destruction of their billboard because Harry was on vacation at the time of the accident. B. Company should not be held liable in negligence because it was Harry's negligence that was the sole proximate cause of Announcement's injuries. C. Company should not be held liable because Announcement has not suffered a harm that is compensable in negligence law. D. Company should be held liable in negligence because one of the risks that made Company negligent was that Harry would drive poorly and cause an accident.

C. Yes, Hotel may be liable for Curtis's injuries despite the industry custom if requiring emergency lighting in hotel rooms is reasonable with respect to the risk of guests injuring themselves during a power outage

Curtis was a guest at Hotel where at 9:00 p.m. one evening there was a power outage that deprived Hotel of electricity. As there was no emergency lighting in his dark room, Curtis fell and injured himself. Curtis sued Hotel for his injuries, alleging that Hotel was negligent in not having inexpensive battery-powered lighting fixtures installed in its guest rooms. Curtis alleges that if his room had such a fixture, he would have avoided injury. The only expert to testify at trial explains that at the time of Curtis's injury, it was not customary in the hotel industry for in-room emergency lighting of this sort to be provided. May Hotel be subject to liability in negligence although if it was in compliance with customary hotel industry standards by not providing in-room emergency lighting? A. No, the evidence of Hotel's compliance with customary hotel industry standards, by not providing in-room emergency lighting, is conclusive proof that Hotel was not negligent. B. Yes, the evidence of Hotel's compliance with customary hotel industry standards is legally irrelevant in determining whether Hotel was negligent. C. Yes, Hotel may be liable for Curtis's injuries despite the industry custom if requiring emergency lighting in hotel rooms is reasonable with respect to the risk of guests injuring themselves during a power outage D. No, Hotel may be liable for Curtis's injuries despite the industry customs because Curtis was injured in one of Hotel's room through no fault of his own.

A. Doctor likely committed a battery.

Doctor while conducting an operation on plaintiff's left eye decides to examine the rest of plaintiff's face and discovers that plaintiff has a deviated nasal septum. At the time of the discovery the plaintiff was under the effect of general anesthesia and unconscious. The doctor decides to correct the deviated septum while the plaintiff rather than do it separately which would require another operation. Which of the following is true? A. Doctor likely committed a battery. B. Doctor will not be liable unless he performed the nasal septum operation negligently. C. Doctor's may assume consent from patients who are under general anesthesia to perform medically warranted procedures. D. Doctor likely committed an assault and battery.

B. No, David just needs to get over it.

David tells a friend that he just got a new job at Blouse Barn. His friend excitedly hugs him to congratulate him. David, while not hurt, exclaims that he's deeply offended because he doesn't like touching. He decides to sue his friend for battery. Will he be successful? A. Yes, battery can occur when there is harmful or offensive contact, so all the elements are there. B. No, David just needs to get over it.

C. Defendant will likely be held liable in negligence because her brakes failed because she unreasonably failed to properly maintain her brakes.

Defendant, driving a car, approaches Plaintiff's car, and properly steps on the brakes. Defendant's brakes fail to function because she carelessly maintained the brakes. Defendant's car continues forward. In this emergency, Defendant chooses to step on the brakes again, and they fail again, causing Defendant's car to strike Plaintiff's car, injuring Plaintiff. Will Defendant likely be held liable for negligence? A. Defendant will likely not be held liable for negligence because in light of her emergency circumstances she behaved as a reasonable prudent person. B. Defendant will likely be held liable in negligence because she should have chosen better by turning the car quickly to the right C. Defendant will likely be held liable in negligence because her brakes failed because she unreasonably failed to properly maintain her brakes. D. Defendant will likely not be held liable in negligence because she was reacting to an emergency situation.

A. Defendant will likely not be held liable for negligence because in light of her emergency circumstances, she behaved as a reasonable prudent person.

Defendant, driving a car, approaches Plaintiff's car, and properly steps on the brakes. For reasons unrelated to any negligence on Defendant's part, her brakes fail to function. Much to her surprise, Defendant's car continues forward, so she quickly pumps the brakes again, but to no avail. Defendant's car careens into Plaintiff's car, injuring Plaintiff. Upon reflecting calmly on Defendant's brake-failing situation, a better choice for Defendant would have been for her to turn her car quickly to the right. In Plaintiff's lawsuit against Defendant in which he seeks to recover from her in negligence, will Defendant likely be held liable in negligence? A. Defendant will likely not be held liable for negligence because in light of her emergency circumstances, she behaved as a reasonable prudent person. B. Defendant will likely be held liable in negligence because she should have chosen more wisely by turning the car quickly to the right. C. Defendant will likely be held liable in negligence if she caused Plaintiff's injuries. D. Defendant will likely not be held liable in negligence because she was reacting to an emergency situation.

C. Doctor owes a duty of reasonable care to Kylie and may be subject to liability for the harm due to Dexter's continuing abuse of her.

Doctor, a clinical psychologist, becomes aware during the course of counseling, that his patient Dexter is sexually abusing Dexter's eight-year-old stepdaughter, Kylie. Doctor does not communicate this information to Kylie's mother or appropriate officials. He also does not take any other steps to prevent Dexter from continuing his sexual assaults of Kylie. When Kylie's mother discovers the abuse, she sues Doctor on behalf of Kylie for his negligence in not telling her or others. Which of the following statements regarding Doctor's liability in negligence for failure to warn is correct? A. Doctor had no affirmative duty in negligence to tell anyone about Dexter or to prevent Dexter from harming Kylie. B. Dexter is the sole proximate cause of the injuries to Kylie, not Doctor. C. Doctor owes a duty of reasonable care to Kylie and may be subject to liability for the harm due to Dexter's continuing abuse of her. D. Doctor may not be held liable for the intentional, intervening, criminal acts of another.

B. Driver would not have hit her or would have caused her less harm if he had not been exceeding the speed limit at the time she was hit.

Driver is driving his car at 60 miles per hour on a stretch of highway with a 50 mile per hour speed limit when he runs into Pedestrian, seriously injuring her. In her negligence action against Driver to recover for her injuries, Pedestrian alleges that Driver was negligent in exceeding the speed limit. In order to establish factual causation (also known as causation-in-fact) as part of her negligence action, what will Pedestrian need to prove? A. Driver ran into her when he was speeding. B. Driver would not have hit her or would have caused her less harm if he had not been exceeding the speed limit at the time she was hit. C. It was foreseeable to Driver that while speeding, he would run into a person walking along the highway thereby causing physical injury. D. It was foreseeable to Driver that while speeding he would run into Pedestrian, causing her injuries.

A. Charles is correct that Drug Co. is a factual cause of Charles's injury, but his negligence action will likely fail for lack of proximate cause.

Drug Co. negligently manufactures and sells a defective contraceptive pill, which fails to prevent Yolanda's pregnancy. Yolanda gives birth to Child A, who grows up and gives birth to Child B. Child B grows up to be a motorcycle enthusiast, who negligently causes a motorcycle accident that injures Charles. Charles seeks to recover from Drug Co. in a negligence action, alleging that Drug Co.'s negligence was a factual cause of Charles's injury. Will Charles's negligence action likely succeed or fail? A. Charles is correct that Drug Co. is a factual cause of Charles's injury, but his negligence action will likely fail for lack of proximate cause. B. Charles is incorrect that Drug Co. is a factual cause of Charles's injury and therefore cannot be subject to liability in negligence to Charles. C. Charles is correct that Drug Co. is a factual cause of Charles's injury, and therefore his negligence action against Drug Co. will likely succeed. D. Charles is incorrect that Drug Co. is a factual cause of Charles's injury, but his negligence action against Drug Co. may still succeed.

C. Farmer may be held strictly liable for the injuries to Neighbor's property because in initiating a controlled burn fire, Farmer was engaged in an abnormally dangerous activity.

Farmer initiates a controlled burn fire to dispose of dry straw spread over his 50 acres of land. For fires of this sort, there are appropriate precautions that must be taken, such as placing various types of obstacles at the property's boundary line. However, even when all reasonable precautions are taken, such fires escape Farmer's property 10% of the time. Because of the size of these fires, when a fire escapes, the damage done to neighboring property is likely to be substantial. Farmer takes all the reasonable precautions to ensure that the fire will remain contained. However, when Farmer's fire is in progress, the wind unexpectedly picks up causing the fire to spread to the property immediately adjoining Farmer's land. Neighbor's property is damaged as a result. In a lawsuit brought by Neighbor against Farmer for property damage caused by the fire, may Farmer be held liable? A. Farmer may be held liable in negligence for the injuries to Neighbor's property because Farmer failed to act as the reasonable prudent person in the circumstances. B. Farmer may not be held liable for the injuries to Neighbor's property because the fire was spread to Neighbor's property by a force majeure or an Act of God and was therefore unforeseeable. C. Farmer may be held strictly liable for the injuries to Neighbor's property because in initiating a controlled burn fire, Farmer was engaged in an abnormally dangerous activity. D. Farmer may not be held liable for the injuries to Neighbor's property because even when all reasonable precautions are taken, such fires escape Farmer's property only 10% of the time.

C. The reasonable prudent person standard will not take into account that Farmer has a lower IQ than most individuals.

Farmer is a country farmer who has a lower IQ than most individuals. However, he is also unusually physically fit, much physically bigger and stronger than the average person. Lately, as part of his trade as a farmer, he has been stacking hay on his property. However, he has been doing so in a way that a farmer of ordinary intelligence would know will cause the hay to combust spontaneously into flames. Defendant's hay does in fact combust causing a fire, which spreads to Neighbor's property causing property damage. In the negligence action brought by Neighbor against Farmer, how should the court instruct the jury regarding the reasonable prudent person standard of care? A. The reasonable prudent person standard will take into account that Farmer has a lower IQ than most individuals. B. The reasonable prudent person standard will not take into account that Farmer was physically bigger and stronger than the average person. C. The reasonable prudent person standard will not take into account that Farmer has a lower IQ than most individuals. D. The reasonable prudent person standard is subjective and based on Farmer's qualities.

A. Daughter may not recover because robber did not know she witnessed the father's beating.

Father was severely beaten by robber and is bringing a claim for assault and battery against robber. Unknown to robber, daughter witnessed father being beaten and suffered emotional distress as a result. If daughter brings a claim for intentional infliction of emotional distress (IIED) against robber, may daughter recover? A. Daughter may not recover because robber did not know she witnessed the father's beating. B. Daughter may recover only if she experienced bodily injury due to the emotional distress. C. Daughter may not recover because she did not suffer bodily injury due to the emotional distress. D. Daughter may recover even if robber was unaware of her presence.

C. The defendant's conduct must be extreme and outrageous AND must cause severe emotional distress.

For intentional infliction of emotional distress claims, which of the following is most accurate? A. The defendant's conduct must be extreme and outrageous AND must cause significant emotional distress. B. The defendant's conduct must be extreme or outrageous AND must cause severe emotional distress. C. The defendant's conduct must be extreme and outrageous AND must cause severe emotional distress. D. None of the above. They are close but not quite right.

B. Even if the factfinder determines that there is no reasonable alternative design available that would provide similar prank characteristics, Funny Business can still be held liable in tort for the cigar having a defective design if the factfinder determines that the utility of the exploding cigar is so low and the risk of injury was so high that the cigar should not have been marketed at all.

Funny Business manufactures novelty items, including an exploding cigar made to blow up with a loud bang and the emission of smoke. Employee purchased the exploding cigar and presented it to Boss at an office birthday party. Boss lit the cigar. When it exploded, the heat from the explosion lit Boss's beard on fire, causing serious burns to his face. In a lawsuit brought by Boss against Funny Business, Boss alleges that he was injured by a defectively designed product. In a products liability action, may Funny Business be held liable? A. If the factfinder determines that there is no reasonable alternative design available that would provide similar prank characteristics, Funny Business cannot be found liable for manufacturing a cigar with a defective design. B. Even if the factfinder determines that there is no reasonable alternative design available that would provide similar prank characteristics, Funny Business can still be held liable in tort for the cigar having a defective design if the factfinder determines that the utility of the exploding cigar is so low and the risk of injury was so high that the cigar should not have been marketed at all. C. Boss will be barred from recovery from Funny Business if his lighting of the cigar was a misuse of the product. D. Boss will not be able to hold Funny Business liable for its exploding cigar because the product did exactly what it was designed to do - it exploded.

A. Gas Co. may be subject to vicarious liability for the negligence of Supply Inc. because Gas Co. hired Supply Inc. to carry out an activity that posed a highly dangerous risk and Supply Inc.'s negligence was a factual cause of Neighbor's harm.

Gas Co. supplies natural gas for the county and hires Supply Inc. to install a service line to carry gas to a new neighborhood. Supply Inc.'s employees negligently cause a slight leak in one of the gas pipes. The resulting leak causes an explosion that injures Neighbor. In a lawsuit brought by Neighbor against Gas Co., may Gas Co. be subject to vicarious or direct liability? A. Gas Co. may be subject to vicarious liability for the negligence of Supply Inc. because Gas Co. hired Supply Inc. to carry out an activity that posed a highly dangerous risk and Supply Inc.'s negligence was a factual cause of Neighbor's harm. B. Gas Co. may not be subject to vicarious liability for the negligence of Supply Inc. because Supply Inc. was an independent contractor. C. Gas Co. may be subject to direct liability for the negligence of Supply Inc. because Gas Co. hired Supply Inc. and Supply Inc.'s negligence was a factual harm of Neighbor's harm. D. Gas Co. may be subject to direct liability for the negligence of Supply Co. because Gas Co. hired Supply Co. to carry out a highly dangerous activity.

A. That Fanny could be injured was foreseeable from George's negligence and therefore George can be subject to liability.

George, a concessionaire at a sports stadium, negligently collided with Fanny, a fan attending the game, knocking her to the ground. Fanny had an asymptomatic herniated disc in her back that results in her suffering serious back injury and pain as a result of George's collision with her. In a negligence action brought by Fanny against George, may George be subject to liability for Fanny's injuries? A. That Fanny could be injured was foreseeable from George's negligence and therefore George can be subject to liability. B. George cannot be held liable for Fanny's injuries because he could not have foreseen that she was suffering from an asymptomatic herniated disc. C. George may be subject to liability for the collision itself, but not for the extensive back injuries because the extensive back injuries were not foreseeable. D. George may only be held liable to Fanny for injuries that he intended to cause.

B. Podiatrist may be liable to Hoda for the loss of her leg because Dr. Primary's negligence was a foreseeable intervening event.

Hoda, a diabetic, required special orthopedic shoes. Podiatrist negligently fitted Hoda with her orthopedic shoes. The improperly fitted shoes caused blistering on Hoda's feet. Hoda saw Dr. Primary about the blistering, but Dr. Primary decided nothing need be done. However, because of her diabetic condition, Hoda developed gangrene in one of her feet and had to have her leg amputated. Hoda sued Podiatrist for his negligence in fitting her with the shoes, seeking to recover damages for the amputation of her leg. Podiatrist contends that if Dr. Primary had not been negligent in failing to take prompt action to treat Hoda's blistering, Hoda would not have lost her leg and Podiatrist therefore should not be liable to Hoda for the loss of her leg. Which of the following statements is correct? A. Podiatrist should not be liable to Hoda for the loss of her leg because Dr. Primary's negligence was a superseding cause. B. Podiatrist may be liable to Hoda for the loss of her leg because Dr. Primary's negligence was a foreseeable intervening event. C. If Podiatrist is liable to Hoda for the loss of her leg, then Dr. Primary cannot also be liable. D. If Dr. Primary is labile to Hoda for the loss of her leg, the Podiatrist cannot also be liable.

A. The bottle that caused Jeremy's harm contained a manufacturing defect.

Jeremy purchased a bottle of liquor from Liquor Barn (not to be confused with Blouse Barn). The liquor was bottled by Bottlers Inc., in bottles manufactured by A#1 Glass Co. While Jeremy was opening the bottle, it suddenly exploded, causing disfiguring cuts to his face. The fact finder determines that, originating with A#1 Glass Co., the bottle contained a weakness in the glass structure that caused Jeremy's harm and was a departure from the product's intended design. The weakness in the bottle's structure caused the bottle to explode. In a lawsuit brought by Jeremy against Liquor Barn, A#1 Glass Co., and Bottlers Inc., what type of defect did the bottle have? A. The bottle that caused Jeremy's harm contained a manufacturing defect. B. The bottle that caused Jeremy's harm contained a design defect. C. The bottle that caused Jeremy's harm contained a warnings defect. D. The bottle that caused Jeremy's harm contained an inadequate instructions defect.

B. Hotel Owner may not be subject to direct liability for the actions of Pipe Co. because Hotel Owner did not owe a duty of care with respect to the manner in which Pipe Co. performed its work and did not retain control over any part of its work.

Hotel Owner contacts Pipe Co. to deliver and install a large pipe to display a sign for its hotel. Hotel Owner thoroughly vets Pipe Co., and Pipe Co.'s representative sends Hotel Owner information showing that Pipe Co. has extensive experience in installing pipes like this and that Pipe Co. is bonded for this purpose. When Pipe Co.'s workers attempt the installation by using their truck, hooks, chains, and additional poles for stabilization, the pipe falls, bouncing on a passing car and injuring Driver. In a lawsuit brought by Driver against Hotel Owner for injuries sustained when the pipe fell on his car, may Hotel Owner be subject to direct or vicarious liability? A. Hotel Owner may be subject to direct liability in negligence because he failed to use reasonable care in hiring Pipe Co. B. Hotel Owner may not be subject to direct liability for the actions of Pipe Co. because Hotel Owner did not owe a duty of care with respect to the manner in which Pipe Co. performed its work and did not retain control over any part of its work. C. Hotel Owner may be subject to vicarious liability in negligence because he hired Pipe Co. and Pipe Co.'s negligence factually caused injuries to Driver. D. Hotel Owner may be subject to vicarious liability for Driver's injuries because employers are responsible for the torts of their independent contractors

A. Hunter may not be held liable in negligence for Katie's broken toe because the risk that makes Hunter negligent is that Katie might shoot something or some person with the gun, not that she would drop it on her toe and hurt herself.

Hunter finishes his day working in the field and stops at a friend's house on his walk home from work. His friend's nine-year-old daughter, Katie, greets Hunter, who hands his loaded shotgun to her as he enters the house. The shotgun is neither especially heavy nor unwieldy. Nonetheless, Katie drops the shotgun, which lands on her toe, breaking it. In a negligence action against Hunter for Katie's broken toe, may Hunter be held liable in negligence? A. Hunter may not be held liable in negligence for Katie's broken toe because the risk that makes Hunter negligent is that Katie might shoot something or some person with the gun, not that she would drop it on her toe and hurt herself. B. Hunter may be held liable in negligence for Katie's broken toe because he negligently provided her with the shotgun. C. Hunter may not be held liable because Hunter's act of giving Katie the gun is not a factual cause of Katie's broken toe. D. Hunter may not be held liable for Katie's broken toe if the manner in which she ended up injuring herself was unforeseeable.

A. Whether the intent element was satisfied

In Garratt v Dailey, what was the issue on appeal? A. Whether the intent element was satisfied B. Whether Brian Dailey was old enough to be held liable at age 5-3/4. C. Whether the contact element was satisfied.

B proof of the plaintiff's comparative fault may operate to reduce the plaintiff's recovery by the percentage of fault attributable to her.

In all jurisdictions that have replaced the traditional defense of contributory negligence with comparative fault: A. comparative fault schemes were adopted judicially. B proof of the plaintiff's comparative fault may operate to reduce the plaintiff's recovery by the percentage of fault attributable to her. C comparative fault schemes were adopted legislatively. D proof of the plaintiff's comparative fault is a complete bar to the plaintiff's recovery.

C. Jeff should not be liable in negligence because Jeff's negligent conduct of speeding did not increase the risk of the type of harm suffered by Nate.

Jeff is driving 35 miles per hour on a city street with a speed limit of 25 miles per hour with Nate as his passenger. But for his speeding, Jeff would not have been at the very place on the road when, without warning, a tree crashes onto Jeff's car, injuring Nate. In a negligence action brought by Nate against Jeff, should Jeff be liable for negligence? A. Jeff should not be liable in negligence because Jeff's speeding is not a factual cause of Nate's harm. B. Jeff should be liable in negligence because Jeff's negligent conduct of speeding increased the risk of the type of harm suffered by Nate. C. Jeff should not be liable in negligence because Jeff's negligent conduct of speeding did not increase the risk of the type of harm suffered by Nate. D. Jeff should not be liable in negligence because a passenger of a car cannot successfully sue the driver of the car in negligence.

C. Liquor Barn, Bottlers Inc., and A#1 Glass Co. may all be subject to liability even though they exercised reasonable care in the preparation and distribution of the defective bottle of liquor.

Jeremy purchased a bottle of liquor from Liquor Barn. The liquor was bottled by Bottlers Inc., in bottles manufactured by A#1 Glass Co. While Jeremy was opening the bottle, it suddenly exploded, causing disfiguring cuts to his face. The fact finder determines that, originating with A#1 Glass Co., the bottle contained a weakness in the glass structure that caused Jeremy's harm and was a departure from the product's intended design. The weakness in the bottle's structure caused the bottle to explode. In a lawsuit brought by Jeremy against Liquor Barn, A#1 Glass Co., and Bottlers Inc., which of the defendants may be subject to liability for the defective product? A. Liquor Barn, Bottlers Inc., and A#1 Glass Co. may all be subject to liability only if they failed to exercise reasonable care in the preparation and distribution of the defective bottle of liquor. B. Only A#1 Glass Co. may be subject to liability because the defective bottle originated with A#1 Glass Co. C. Liquor Barn, Bottlers Inc., and A#1 Glass Co. may all be subject to liability even though they exercised reasonable care in the preparation and distribution of the defective bottle of liquor. D. A#1 Glass Co. may be subject to liability only if it can be established that A#1 Glass Co. failed to exercise reasonable care in the preparation of the defective bottle.

D. Jill may be held liable to Pedestrian in negligence because the reasonable prudent person would likely not have lent her car to Friend, knowing that Friend's license was suspended for repeated instances of reckless driving.

Jill loans her car for the evening to Friend who needs the car for social purposes. Jill knows that Friend's driver's license was suspended a month previously on account of repeated instances of reckless driving. In the course of the evening, Friend drives the car negligently and injures Pedestrian. In a cause of action against Jill brought by Pedestrian for recovery of her injuries, can Jill be held liable for negligence? A. Jill cannot be held liable in negligence because Friend was driving the car and he was therefore the sole proximate cause for Pedestrian's injuries. B. Jill cannot be held liable in negligence because an adult cannot be held liable for the tortious conduct of another adult. C. Jill may be held liable to Pedestrian in negligence because car owners are responsible for injuries caused by their vehicles. D. Jill may be held liable to Pedestrian in negligence because the reasonable prudent person would likely not have lent her car to Friend, knowing that Friend's license was suspended for repeated instances of reckless driving.

A. Waiter may not be subject to liability in negligence because Waiter had no affirmative duty to render aid or call for assistance for Joe.

Joe has just finished eating his lunch and is walking alone down the sidewalk in front of a restaurant when he suddenly suffers a severe asthma attack. Waiter, who works at the restaurant but is on a break at the time, sees Joe through a window and realizes that Joe is suffering an asthma attack. Nonetheless, Waiter chooses to do nothing to help Joe. As a result, the appropriate medical care for Joe is delayed, causing Joe serious physical injuries that could have otherwise been avoided. Joe sues Waiter in negligence for his injuries that result from the delay in his receiving medical care. May Waiter be liable to Joe in negligence? A. Waiter may not be subject to liability in negligence because Waiter had no affirmative duty to render aid or call for assistance for Joe. B. Waiter may be subject to liability in negligence for failing to render aid or call for assistance for Joe because Waiter realized that Joe was in need of prompt medical attention. C. Waiter may be subject to liability in negligence for failing to render aid or call for assistance for Joe, but only if Waiter knew that no one else was present to call for assistance. D. Waiter cannot be subject to liability in negligence because Waiter's failure to act was not the factual cause of Joe's injuries.

B. Ladder Co. is likely to be found not liable as a matter of law because a product seller is not subject to liability for failing to warn or instruct regarding risk that should be obvious to or generally known by foreseeable product users.

Ladder Co. manufactures kitchen stepladders for residential use. Dad used a Ladder Co. ladder to post a sign above the door of his home office, unaware that his five-year-old son was playing in the office. While Dad was standing on the ladder, his son suddenly opened the door, striking the ladder. Dad fell off the ladder, fracturing his hip. The ladder came with no warnings about not using it in front of an unlocked door. In a lawsuit brought by Dad against Ladder Co. for products liability, is Ladder Co. likely to be found liable? A. Ladder Co. is likely to be found liable because the ladder it manufactured was a factual cause of Dad's injury. B. Ladder Co. is likely to be found not liable as a matter of law because a product seller is not subject to liability for failing to warn or instruct regarding risk that should be obvious to or generally known by foreseeable product users. C. Ladder Co. is likely to be found liable because it failed to warn of the risk that resulted in the injury of Dad. D. Ladder Co. is only liable to Dad if a reasonable person would have stood on the ladder as he did.

C. Jacob's trespass claim will succeed, but Marty's assault claim will fail.

Marty and Jacob are neighbors. Jacob likes to listen to music loud in his backyard, and it's driving Marty crazy. One night, Marty decides to cross over to Jacob's yard and cut all the wiring to the outdoor sound system in Jacob's yard. As he's tiptoeing across the yard, he starts to have second thoughts, so he carefully retraces his steps backward, making sure not to cause any damage. He even goes so far as to fluff up the blades of grass where he made footprints. When he's 8 feet away from being safely back on his property, Jacob spots him from an upstairs window, opens the window, and yells, "If you don't get off my property, I'll kill you." Marty, a former long jumper, takes one leap back and lands on his own property, tumbles to the ground, gets up, and races into his house. Which statement is most accurate? A. Marty's assault claim will fail, and Jacob's trespass claim will fail. B. Marty's assault claim will succeed, but Jacob's trespass claim will fail. C. Jacob's trespass claim will succeed, but Marty's assault claim will fail. D. Marty's assault claim will succeed, and Jacob's trespass claim will succeed.

C. Due to the current privity rules, Marty can sue Acme, Buzzer, or both.

Marty purchased a Buzzer lawnmower from Acme Hardware. The first time he used it to mow his lawn, the blade guard intended to protect against injury popped off, and the blade cut off two of Marty's toes. Which of the following is the most accurate statement? A. Due to privity requirements, Marty can sue only Acme. B. Due to the current privity rules, Marty must sue Acme first, but if unsuccessful, he can then sue Buzzer. C. Due to the current privity rules, Marty can sue Acme, Buzzer, or both. D. Whatever happens, Marty won't recover the full amount of his losses, as he was partly at fault and under a comparative negligence regime, his recovery will be reduced.

D. Max may be held directly liable for Customer's injuries because the injuries for which Customer seeks recovery are foreseeable from Max's negligence in not conducting a background check.

Max owns a furniture store and employs Alex to deliver furniture to retail customers. Alex's duties include entering customers' homes to situate items they have purchased. Having entered Customer's home to deliver a sofa, Alex assaults Customer. Prior to employing Alex, Max conducted no check of Alex's background, although a reasonable employer of an employee who would be entering the homes of customers would have conducted such a check. Had Max done so, Max would have discovered criminal convictions for assault and would not have employed Alex to make deliveries. Customer seeks to hold Max directly liable in negligence for Customer's injuries. Can Max be held directly liable for Customer's injuries? A. Max cannot be held directly liable for Customer's injuries because an employer cannot be held liable for the intentional tortious acts of its employees. B. Max cannot be held directly liable for Customer's injuries because an employer is not responsible for the criminal acts of its employees. C. Max may be held directly liable in negligence because employers are always responsible for the wrongful conduct of their employees. D. Max may be held directly liable for Customer's injuries because the injuries for which Customer seeks recovery are foreseeable from Max's negligence in not conducting a background check.

C. Mom is likely not liable in negligence if turning her back for that brief period of time was reasonable.

Mom is the mother of Toddler, who is almost two years old. Mom and Toddler are visiting Friend's quaint rustic cabin. One morning when Friend has gone to run errands, Mom and Toddler are in the kitchen, a lovely room lit by a kerosene lamp sitting on a kitchen table. Toddler knocks the lantern over during a less than 10-second period during which Mom has turned her back in order to take a boiling pot off the stove. The knocked over lantern starts a fire that damages Friend's cabin. In a negligence action brought by Friend against Mom, is Mom likely to be held liable in negligence? A. Mom is likely not to be held liable in negligence because Toddler caused the fire, not her. B. Mom is likely liable for negligence if she failed to act as the reasonable prudent person when she left Toddler alone next to a kerosene lamp. C. Mom is likely not liable in negligence if turning her back for that brief period of time was reasonable. D. Mom is likely liable in negligence because parents are vicariously responsible for the damage caused by their children.

B. Store clerk would likely not recover because the shoppers comment is legally insufficient to support a cause of action for IIED.

Shopper insulted store clerk by saying that the store clerk had a disgusting body odor. Shopper intended to insult and embarrass store clerk and said it loud enough for the fellow shoppers in the vicinity to hear. Store clerk was mortified when other shoppers laughed. As a result of the insult, store clerk fainted and hit his head on a display and suffered a concussion. If store clerk brought an action for Intentional Infliction of Emotional Distress against shopper, would store clerk recover? A. Store clerk would recover because he intended to cause emotional harm and did in fact cause emotional harm. B. Store clerk would likely not recover because the shoppers comment is legally insufficient to support a cause of action for IIED. C. Store clerk would recover if the harm suffered was indeed severe. D. Store clerk would not recover if he is considered the equivalent of an innkeeper.

D. Mom is likely to be held liable in negligence to Friend because Mom failed to act as the reasonable prudent person when she left Toddler alone for an hour next to a kerosene lamp.

Mom is the mother of Toddler, who is almost two years old. Mom and Toddler are visiting Friend's quaint rustic cabin. One morning when Friend has gone to run errands, Mom and Toddler are in the kitchen, a lovely room lit by a kerosene lamp sitting on a kitchen table. When Mom leaves the kitchen for an hour to read a book, Toddler knocks over the lantern, starting a fire that damages Friend's cabin. In a negligence action brought by Friend against Mom, is Mom likely to be held liable in negligence? A. Mom is not likely to be held liable to Friend in negligence because Toddler started the fire, not Mom. B. Mom is not likely to be held liable in negligence to Friend because Friend assumed the risk by his ownership of the cabin. C. Mom is likely to be held liable in negligence because parents are vicariously responsible for the damage caused by their children. D. Mom is likely to be held liable in negligence to Friend because Mom failed to act as the reasonable prudent person when she left Toddler alone for an hour next to a kerosene lamp.

C. Despite her exercise of reasonable care in attempting to confine the lion, Nala may be held strictly liable to Neighbor for his injuries.

Nala keeps a young lion in her backyard in order to scare off any intruders. The lion is carefully fastened to a post by a chain. Because of a defect in the chain that Nala could not be expected to detect, the lion breaks free and mauls Neighbor on the sidewalk, injuring him. In a lawsuit brought by Neighbor against Nala, may Nala be subject to liability? A. Because Nala exercised all due care in confining the lion, Nala may not be subject to liability to Neighbor for his injuries. B. Nala may not be held liable for the injuries to Neighbor because she could not have been expected to detect the defect in the chain. C. Despite her exercise of reasonable care in attempting to confine the lion, Nala may be held strictly liable to Neighbor for his injuries. D. Nala may be subject to liability in negligence for the injuries her lion cased to Neighbor.

true

The basic rule of nonfeasance is that the defendant has no duty to act in an affirmative way unless the defendant is in a special relationship with the plaintiff or with the other person who threatens the harm. True False

A. Each of Rosie's and Vincent's negligence is a factual cause of the destruction of Company's hunting lodge.

Rosie and Vincent do not know each other and were camping independently in a heavily forested campground. Each one had a campfire, and each one negligently failed to ensure that the fire was extinguished upon retiring for the night. Due to unusually dry forest conditions and a stiff wind, both campfires escaped their sites. The two fires, burning out of control, eventually joined together and engulfed Company's hunting lodge, destroying it. Either fire alone would have destroyed the lodge. In a lawsuit brought by Company against both Rosie and Vincent alleging negligence, which of the following statements is correct regarding factual causation? A. Each of Rosie's and Vincent's negligence is a factual cause of the destruction of Company's hunting lodge. B. Neither Rosie's nor Vincent's negligence is a factual cause of the destruction of Company's hunting lodge. C. Rosie's negligence is the sole factual cause of the destruction of Company's hunting lodge. D. Vincent's negligence is the sole factual cause of the destruction of Company's hunting lodge.

A. No, because the threat by coworker was not one which required immediate action.

On Friday, employee is credibly threatened by coworker. Coworker says to employee, "I will kill you next Friday if you show up to work then." Employee and three friend's go to Coworker's house the day after Coworker threatened employee and beat Coworker to such an extent that the Coworker will be hospitalized long term and incapable of carrying out his threat. May the employee successfully assert the privilege of self-defense in a battery suit by Coworker against Employee? A. No, because the threat by coworker was not one which required immediate action. B. Yes, because Coworker credibly threatened employee with death so employee could use up to lethal force in defending himself. C. Yes, because employee can preemptively prevent the threat from materializing. D. No, because the person asserting self-defense must wait for the attacker to begin attacking before defending.

D. Darcy's recovery in negligence will be barred if the defendant can prove that Darcy voluntarily encountered the risk of being hit in the mouth by a hockey puck and was aware that she might have her front teeth knocked out.

One day Darcy, a high school teacher, takes her students to watch an ice hockey game starring the local semi-professional team. Aware that the students are in attendance at the game that day, the team owner invites Darcy and the kids onto the ice during intermission. As Darcy steps onto the ice, a hockey puck flies through the air, hitting her in the face and knocking out her front teeth. Darcy sues the team in negligence. In a jurisdiction that recognizes the traditional defense of assumption of the risk, what effect will the defense have on Darcy's claim? A. Darcy's recovery in negligence will be reduced by the percentage of fault attributable to her if the defendant can prove that Darcy voluntarily encountered the risk of being hit in the mouth by a hockey puck and was aware that she might have her front teeth knocked out. B. Darcy's recovery in negligence will be reduced by the percentage of fault attributable to her if the defendant can prove that a reasonable person would not have voluntarily encountered the risk of being hit in the mouth by a hockey puck. C. Darcy's recovery in negligence will be barred if the defendant can prove that a reasonable person would not have voluntarily encountered the risk of being hit in the mouth by a hockey puck. D. Darcy's recovery in negligence will be barred if the defendant can prove that Darcy voluntarily encountered the risk of being hit in the mouth by a hockey puck and was aware that she might have her front teeth knocked out.

false

One lazy Sunday, after having too much to drink, Aaron went to the Stop & Shop supermarket to buy snacks. As he walked (or rather, weaved) up and down the aisles, he came to the juice section. Too drunk to steadily take what he wanted from the shelves, Andy knocked over a gallon bottle of apple juice, which crashed to the ground, shattered, sending juice spilling across the aisle. Andy quickly, albeit awkwardly, scampered away. Ten minutes later, Wallace walked down the aisle, pushing a cart full of groceries with his two-year-old son in the child seat of the cart. With all that was piled up in his cart, Wallace couldn't see the spilled juice. He slipped and ended up with a concussion and a broken wrist. He sued the grocery store. During the trial, it was discovered that industry standard recommends that an employee walk the aisles every 15 minutes to identify and correct any hazards, which Stop & Shop complied with. Miriam, an employee, saw the spill occur and reported it to Nancy who added it to the list of issues to address during the next walk through of the aisles. Unfortunately, Wallace was injured before the next walk-through occurred. If Miriam had not seen the spill (and thus the store did not know about it), the lawsuit would have been dismissed. True False

yes

Ruth Garratt goes to hospital for hip fracture and gets evaluated. In addition to saying she will fix Ruth's hip, the doctor recommends removal of precancerous moles on Ruth's arm. Ruth says no, but the doctor does it anyways while operating on hip. Will the doctor be liable for battery? Yes No

B. The elderly man can sue for battery and will be successful.

Sandy, 17, throws a snowball at a friend on a crowded street corner. The snowball missed the friend but hits an elderly man who was walking by at that moment. The man falls to the ground and is injured. Which of the following best describes the most likely outcome? A. The elderly man can sue for assault and will be successful. B. The elderly man can sue for battery and will be successful. C. The elderly man can sue for intentional infliction of emotional distress and will be successful. D. None of the above.

C. Wallace's lawsuit will be successful because the store had notice of a hazard and failed to correct it in a timely manner.

One lazy Sunday, after having too much to drink, Aaron went to the Stop & Shop supermarket to buy snacks. As he walked (or rather, weaved) up and down the aisles, he came to the juice section. Too drunk to steadily take what he wanted from the shelves, Andy knocked over a gallon bottle of apple juice, which crashed to the ground, shattered, sending juice spilling across the aisle. Andy quickly, albeit awkwardly, scampered away. Ten minutes later, Wallace walked down the aisle, pushing a cart full of groceries with his two-year-old son in the child seat of the cart. With all that was piled up in his cart, Wallace couldn't see the spilled juice. He slipped and ended up with a concussion and a broken wrist. He sued the grocery store. During the trial, it was discovered that industry standard recommends that an employee walk the aisles every 15 minutes to identify and correct any hazards, which Stop & Shop complied with. Miriam, an employee, saw the spill occur and reported it to Nancy who added it to the list of issues to address during the next walk through of the aisles. Unfortunately, Wallace was injured before the next walk-through occurred. 8. Which is the most accurate description of the likely outcome in this case: A. Wallace's lawsuit will not be successful, because Stop & Shop followed industry custom. B. Wallace's lawsuit will be successful because although they followed industry custom, the custom was not reasonable. C. Wallace's lawsuit will be successful because the store had notice of a hazard and failed to correct it in a timely manner. D Wallace's lawsuit will not be successful because the store did not act negligently—it had notice but it did not have sufficient time to correct the hazard.

false

One lazy Sunday, after having too much to drink, Aaron went to the Stop & Shop supermarket to buy snacks. As he walked (or rather, weaved) up and down the aisles, he came to the juice section. Too drunk to steadily take what he wanted from the shelves, Andy knocked over a gallon bottle of apple juice, which crashed to the ground, shattered, sending juice spilling across the aisle. Andy quickly, albeit awkwardly, scampered away. Ten minutes later, Wallace walked down the aisle, pushing a cart full of groceries with his two-year-old son in the child seat of the cart. With all that was piled up in his cart, Wallace couldn't see the spilled juice. He slipped and ended up with a concussion and a broken wrist. He sued the grocery store. During the trial, it was discovered that industry standard recommends that an employee walk the aisles every 15 minutes to identify and correct any hazards, which Stop & Shop complied with. Miriam, an employee, saw the spill occur and reported it to Nancy who added it to the list of issues to address during the next walk through of the aisles. Unfortunately, Wallace was injured before the next walk-through occurred. True or False. Stop & Shop will not be liable because they did not cause the harm. True False

true

One notable difference between dram shop statutes and other regulations governing alcohol is that the former creates a private right of action. True False

C. During the surgery, the doctor realizes the patient suffered from a condition that typically rapidly spreads to all parts of the body and only a careful and painful examination of the patient's entire body would reveal whether or not other parts of the body were affected needing immediate treatment.

Patient gives consent to be treated by surgeon for a minor surgery on plaintiff's left elbow. While plaintiff is unconscious from the anesthetic and during the course of the surgery on the left elbow the surgeon decides to examine the patient's entire body. Surgeon finds nothing else wrong with patient. Patient sues the doctor in battery. What is the best defense of the doctor? A. A patient who consents to surgical treatment on one part of their body automatically consents to examination and treatment of their entire body. B. Because of superior knowledge a treating physician may examine and touch any part of an unconscious patient's body. C. During the surgery, the doctor realizes the patient suffered from a condition that typically rapidly spreads to all parts of the body and only a careful and painful examination of the patient's entire body would reveal whether or not other parts of the body were affected needing immediate treatment. D. The doctor felt that it would save time to examine the patient at this instant rather than schedule a separate appointment.

C. Exclusion is never false imprisonment.

Patron was trying to enter a large exclusive hotel in the middle of a city. The representatives of the hotel met patron at the door and casually informed him that he was not permitted to be on the hotel property. After that, they forcibly excluded him from the hotel property every time he tried to enter. If patron files a false imprisonment claim against the hotel what is the hotel's best defense? A. False imprisonment can never occur on private property. B. False imprisonment cannot occur because property peaceably informed patron he would be a trespasser the first time patron tried to enter. C. Exclusion is never false imprisonment. D. The remainder of the city other than the hotel was too large to be considered false imprisonment.

B. If Rusty has been comparatively negligent, his recovery in strict liability will be reduced according to his share of comparative responsibility.

Petula lives in a residential neighborhood in State and keeps a Bengal tiger in her backyard as a pet. Rusty, Petula's good friend, decides to visit her Bengal tiger. Not realizing how dangerous the big cat can be, Rusty carelessly walks right up to the cage and puts his hand into the enclosure, taunting the animal. The animal becomes agitated, manages to escape from the enclosure, and mauls Rusty. Rusty then sues Petula in strict liability for the injuries he sustained as a result of being mauled by the tiger. Petula defends by claiming that Rusty was comparatively negligent. In a pure comparative fault jurisdiction, which of the following statements is correct? A. Rusty will be able to recover from Petula because comparative negligence is not a defense to a claim of strict liability. B. If Rusty has been comparatively negligent, his recovery in strict liability will be reduced according to his share of comparative responsibility. C. Rusty will not be able to recover from Petula if he was comparatively negligent because comparative negligence is a complete bar to recovery. D. In order to successfully defend this strict liability action, Petula will have to prove that Rusty assumed the risk.

C. The court should deny directed verdict in favor of Pharm Co. because the trier of fact needs to determine whether Pharm Co. should be subject to liability for marketing and distributing NoAcne based on the drug not being reasonably safe due to inadequate instructions.

Pharm Co. manufactures and distributes prescription medicine. After many years of careful research and testing and after receiving approval from the FDA, Pharm Co. released NoAcne, a drug developed to help adolescents combat severe acne. Three years later, Teen, who was 13 years old at the time, received a doctor's prescription and began taking NoAcne for his terrible skin problems. He took NoAcne for approximately one year as prescribed. Two years before Teen began taking NoAcne, medical researchers published a study in a leading medical publication explaining that drugs with similar toxicological effects to NoAcne cause vision problems in teens. Pharm Co. was unaware of the report and did not conduct further research or provide warnings to doctors about the potential for loss of vision in teens using NoAcne. In a failure to warn lawsuit brought on behalf of Teen against Pharm Co., Teen's expert testifies that the reasonable manufacturer would have conducted further testing into the effects of NoAcne and as a result would have warned prescribing physicians of the potential for vision loss in teens. Pharm Co. moves for directed verdict. How should the court rule? A. The court should direct verdict in favor of Pharm Co. because Pharm Co. did not actually know of the possibility of vision loss in adolescents. B. The court should direct verdict in favor of Pharm Co. because Pharm Co. had FDA approval to market and distribute NoAcne. C. The court should deny directed verdict in favor of Pharm Co. because the trier of fact needs to determine whether Pharm Co. should be subject to liability for marketing and distributing NoAcne based on the drug not being reasonably safe due to inadequate instructions. D. The court should deny directed verdict in favor of Pharm Co. because Pharm Co. should be absolutely liable for injuries caused by medication it has manufactured and distributed.

B. Phone Co. may be subject to direct liability to Tony for failing to use reasonable care in selecting a competent contractor.

Phone Co. lays underground cable throughout City. Phone Co. hires Indep as the independent contractor to perform the digging necessary for laying the cable. When the job starts, Indep has no previous digging or construction experience and only several hours of practice using a trenching machine. Prior to hiring Indep, Phone Co. did not inquire into Indep's experience using trenching machines or working around gas pipelines, the presence of which Phone Co. was reasonably certain to arise. During the course of the digging, Indep negligently strikes a gas line, causing an explosion that seriously injuries Tony. In a negligence action brought by Tony against Phone Co., will Phone Co. be subject to liability? A. Phone Co. cannot be subject to liability to Tony because Indep was hired as an independent contractor. B. Phone Co. may be subject to direct liability to Tony for failing to use reasonable care in selecting a competent contractor. C. Phone Co. is subject to liability to Tony for failing to use reasonable care in digging around the gas pipeline because employers are vicariously liable for the tortious conduct of their independent contractors. D. Phone Co. may be subject to vicarious liability to Tony for failing to use reasonable care in selecting a competent contractor.

false

Physical restraint or a physical barrier is required for a successful claim of false imprisonment. True False

B. Moira can successfully sue Roland for battery.

Roland sneaks up behind Johnny who does not see or hear Roland. Just as Roland swings his fist at Johnny, Johnny sneezes, causing his head to bow. As a result, Roland misses Johnny and ends up punching Moira in the nose. What's the result? A. Johnny can successfully sue Roland for battery. B. Moira can successfully sue Roland for battery . C. Roland got lucky. He didn't intend to hit Moira, and he never made contact with Johnny. So he cannot be successfully sued by either.

B. Soda Company may be held vicariously liable for Delivery Person's negligence because Delivery Person was an employee at the time of the accident with Teenager.

Soda Company, a company that provides beverages to local businesses, employs Delivery Person and has provided Delivery Person with detailed instructions on exactly how to conduct its business throughout the workday. Delivery Person is delivering beverages to SuperStore. Teenager is shopping at SuperStore when, while hauling sodas into SuperStore, Delivery Person carelessly runs over Teenager's foot, spraining and bruising it. In a negligence action brought against Soda Company on Teenager's behalf for the injuries sustained when Delivery Person ran over his foot, may Soda Company be held vicariously liable? A. Soda Company may not be held vicariously liable for Delivery Person's negligence because Delivery Person was an independent contractor at the time of the accident. B. Soda Company may be held vicariously liable for Delivery Person's negligence because Delivery Person was an employee at the time of the accident with Teenager. C. Soda Company may not be held vicariously liable for Delivery Person's negligence if the detailed instructions given to Delivery Person made clear that Delivery Person was not to run over anyone's foot while on a delivery. D. Soda Company may be held vicariously liable for Delivery Person's negligence if Delivery Person was on a frolic.

True

Substantial certainty that a harmful or offensive contact will result is sufficient to satisfy the element of intent. True False

B. Brian will avoid liability.

What if Brian's sister pushed him into the chair and it was knocked aside and then Ruth tried to sit and hit the ground instead? A. Tough luck, but little Brian will be liable for battery. B. Brian will avoid liability.

B. If a doctor does not obtain consent, patient can sue for battery but not negligence.

Which of the follow is the best answer? A. If a doctor does not obtain consent, patient can sue for negligence but not battery. B. If a doctor does not obtain consent, patient can sue for battery but not negligence. C. If a doctor does not obtain consent, patient can sue for battery and negligence. D. None of the above.

C. Neither of the above.

Which of the following is a correct rule statement for battery? A. Battery is the infliction of harmful contact. B. Battery is when you intend to make contact with another person and you actually make contact and that contact hurts the other person. C. Neither of the above.

C. If you are related to the plaintiff, you have a duty.

Which of the following is not an accurate description of an exception to the no duty rule? A. Beginning to assist creates a duty. B. Special relationships, such as landlord-tenant, can create a duty. C. If you are related to the plaintiff, you have a duty. D. If you create a risk and fail to mitigate it, then you might be liable.

B. Still an assault. Fear is irrelevant.

Think about the Cullison v. Medley case. What if Cullison was a 438-lb Sumo wrestler, and Ernest was a so-called 98-pound weakling who entered Cullison's place and told him to stay away from my daughter? Assuming the other facts are the same, what's the result? A. No assault there is no way that a huge Sumo wrestler would have fear of imminent harmful or offensive contact from a 98-pounder. B. Still an assault. Fear is irrelevant. C. Ernest could sue Cullison for assault if/when he stood up, because a Sumo wrestler would scare any reasonable person and consequences are an objective standard. D. Ernest would recognize Cullison as his favorite Sumo wrestler of all time and ask to take a selfie with him.

A. Whether the warnings were inadequate is a question of fact.

Ticky Tack Co. manufactures a chemical glue used in do-it-yourself home improvement projects. Susie purchased the glue from Home Store to lay faux wood in her kitchen, which she has been remodeling. In large, capital letters on the front of the container of glue was a warning stating that the fumes of the glue were highly flammable and toxic and that the glue should only be used in a place with adequate ventilation. The box warnings also provided that all sources of fire should be extinguished. Susie opened the windows in her kitchen, but failed to extinguish the pilot light in her gas stove. Midway through laying the tile, the pilot light suddenly ignited the fumes from the glue, and Susie was seriously burned as a result. In a products liability action against Ticky Tack, Susie contends that the warnings were inadequate in failing to state specifically that gas stove pilot lights should be extinguished. Which statement regarding the adequacy of the warnings in a products liability action is correct? A. Whether the warnings were inadequate is a question of fact. B. Whether the warnings were inadequate is a question of law. C. Whether the warnings were inadequate is legally immaterial to Susie's products liability action. D. Without regard to whether the warnings were inadequate or not, Susie cannot hold Ticky Tack liable for her injuries because she purchased the bonding from Home Store, which means that only Home Store can be subject to liability.

A. was a complete bar to the plaintiff's recovery.

Traditionally, contributory negligence as a defense to a negligence action: A. was a complete bar to the plaintiff's recovery. B. was synonymous with an immunity. C. reduced the plaintiff's recovery in appropriate circumstances. D. was synonymous with comparative fault.

false

True or False. Assault is a battery that was not completed. True False

C. Yes because not throwing the preservers to Victim was a substantial factor in bringing about Victim drowning.

Visitor was a customer of Barge, when while on the deck of Barge he accidentally fell into the water as the barge approached the pier. Employee of the barge hears Visitor's cries as he yells from the water that he cannot swim. Employee threw a heavy line (used to tie the barge to the pier) in the direction of the cries. The line came within two feet of the spot of Visitor, but Visitor did not grab the lines. Visitor was carried away from the pier by the outgoing tide and downed. At the time of this incident, the employee was aware that several life preservers were on a rack located near the front of the barge where Visitor fell overboard. The preservers remained in the rack and were not used. If the preservers had been used, Victim probably would not have drowned. Visitor's estate sues Barge in negligence, alleging that the failure to use the life preservers was the factual cause of Victim's drowning. Is Visitor's estate correct that Barge's failure to use the life preservers was a factual cause of Victim's drowning? A. No because the failure to use the life preservers is an omission and an omission cannot be the basis for a negligence action. B. No because it is possible that Victim would have drowned even if the life preservers had been thrown out to him. C. Yes because not throwing the preservers to Victim was a substantial factor in bringing about Victim drowning. D. Yes because Victim drowned when he was a customer of Barge.

B. Motive is immaterial. No change to the result.

What if Brian Dailey was having fun and thought that Ruth Garratt would like falling down (he does this with his sister and she always laughs)? A. The lack of motive to harm or offend would negate the intent element. B. Motive is immaterial. No change to the result.


Conjuntos de estudio relacionados

Honors World History Slear-Final Exam prep

View Set

Family and Consumer Science Composite

View Set

EA 150 Chapter 4 Review Questions

View Set

Chapter 3- cardiac emergencies (Part 2 -

View Set

RN Adult Medical Surgical Online Practice 2019 B

View Set

Module 8: Pharmacology and Intravenous Therapies

View Set